Вы находитесь на странице: 1из 21

ACE

Engineering Academy
Hyderabad | Delhi | Bhopal |Pune | Bhubaneswar | Bengaluru | Lucknow | Patna | Chennai | Vijayawada | Visakhapatnam | Tirupati
H.O: 204, II Floor, Rahman Plaza, Opp. Methodist School, Abids, Hyderabad-500001.
Ph: 040-23234418, 040 -23234419 , 040-23234420, 040-24750437
EE: Electrical Engineering

General Aptitude: term "bug" actually refers to a "particular"


One Mark Questions (Q01 – 05) group of insects.

01. The cost of 7 pens, 8 pencils and 3 03. Sentence improvement:


sharpeners is Rs 20. The cost of 3 pencils, 4 Underestimating its value, breakfast is a
sharpeners and 5 erasers is Rs 21. The cost meal many people skip.
of 4 pens, 4 sharpeners and 6 erasers is Rs (a) Underestimating its value, breakfast is a
25. The cost of 1 pen, 1 pencil, 1 sharpener meal many people skip
and 1 eraser is ________ (Rs) (b) Breakfast is skipped by many people
Ans: 6 because of their underestimating its
Sol: Let the costs of pens, pencil, eraser and value
sharpener be pn, pp, e and s respectively (c) Many people, underestimating the value
Given of breakfast, and skipping it.
7pn + 8pp + 3s = 20 (d) Many people skip breakfast because they
3pp + 4s + 5e = 21 underestimate its value.
4pn + 4s + 6e = 25 Ans: (d)
Adding all three equations Sol: The problem with this sentence is that the
11pn + 11pp + 11s + 11e = 66 opening phrase "underestimating its value"
 1pn + 1 pp + 1s + 1e = 6 modifies "breakfast," not "people." The
order of the words in the sentence in choice
02. Sentence Completion: (d) does not have this problem of a
Although some think the terms "bug" and misplaced modifying phrase. Choice (d) also
"insect" are -------, the former term actually clarifies the causal relationship between the
refers to ------- group of insects. two clauses in the sentence. None of the
(a) parallel - an identical other choices convey the information
(b) precise - an exact presented in the sentence as effectively and
(c) interchangeable - particular directly as choice (d).
(d) exclusive - a separate.
Ans: (c) 04. Spot the error, if any:
Sol: The word "although" indicates that the two If I were her / I would accept / his offer
parts of the sentence contrast with each (a) If I were her,
other: although most people think about the (b) I would accept
terms "bug" and "insect" one way, (c) his offer
something else is actually true about the (d) No error
terms. Choice (c) logically completes the Ans: (a)
sentence, indicating that while most people
think the terms are "interchangeable," the

ACE Engineering Academy Hyderabad |Delhi |Bhopal |Pune |Bhubaneswar |Bengaluru |Lucknow |Patna |Chennai | Vijayawada |Visakhapatnam | Tirupati
:2: Electrical Engg.

Sol: Rule we should use Subjective case of 1


pronoun after BE forms…am, is, are was Putting x = we get
7
were.,, has been, have been, had been. 1 + 22x + 32x2 + 42x3 + 52x4 + - - - - -
Her is an objective case --- s = 1 + 4x + 9x2 + 16x3 + 25x4
If I were she. is correct. s.x = x + 4x2 + 9x3 + 16x4 + - - - - -
s – sx = 1 + 3x + 5x2 + 7x3 + 9x4 + - - - - - -
05. Kishenkant walks 10 kilometres towards x(s – sx) = x + 3x2 + 5x3 + 7x4 + - - - - - -
North. From there, he walks 6 kilometres (s – sx) –x(s – sx) = 1 + 2x + 2x2 + 2x3 + - -
towards south. Then, he walks 3 kilometres - - - - + to 
towards east. How far and in which direction 2x
is he with reference to his starting point? (1 – x)2 s = 1+ ; since x  1
(a) 5 kilometres, West Direction 1 x
(b) 5 kilometres, North-East Direction 1 x
s=
(c) 7 kilometres, East Direction (1  x ) 3
(d) 7 kilometres, West Direction We may use it as direct formula for solving
Ans: (b) this type of problem
Sol: The movements of Kishenkant are as shown 1
Substituting x = we get
in figure 7
B
1
6 km 3 km 1
C D
s= 7 = 8  343  49 =1.81
10 km
 1
3
7  216 27
4 km 1  
 7
A
x y z
07. If    5 and a, b
A to B, B to C and C to D 3a  2b 3b  2c 3c  2a
AC = (AB – BC) = (10 – 6) km = 4 km and c are in continued proportion and b, c, a
Clearly, D is to the North-East of A are in continued proportion, then
 Kishenkant’s distance from starting point A x y z
  is _______ ( a, b and c are in
AD = AC2  CD 2 a 2b 3c
 (4) 2  (3) 2  25 = 5 km continued proportion means b2 = ac)
1
So, Kishenkant is 5 km to the North-East of (a) 55 (b) 25
his starting point 5
1 5
(c) 4 (d) 45
Two Mark Questions (Q06 – 10) 6 6
Ans: (d)
4 9 16 25 Sol: Given that a, b, c are in continued proportion
06. The infinite sum 1+ + 2 + 3 + 4 + - - - -
7 7 7 7  b2 = ac -------- (1)
- equals Also b, c, a are in continued proportion
 c2 = ab ------- (2)
Ans: 1.8 to 2 From (1) and (2)
Sol: We have to find the sum of the series b2c2 = a2bc  a2 = bc ------- (3)
4 9 16 25 Conditions (1), (2) and (3) can only be
1+ + 2 + 3 + 4 + - - - - - satisfied when a = b = c = k (say)
7 7 7 7

ACE Engineering Academy Hyderabad |Delhi |Bhopal |Pune |Bhubaneswar |Bengaluru |Lucknow |Patna |Chennai | Vijayawada |Visakhapatnam | Tirupati
:3: Pre GATE Question Paper with Sol.

x y z x y z (b) Some natural substances found in foods


    5     25 can be harmful
5k 5k 5k k k k
x y z x 1 y 1z (c) Food without additives is unlikely to
      taste good
a 2b 3c k 2 k 3 k (d) Munchon Foods produces only breakfast
25 25
= 25   cereals
2 3 Ans: (b)
25  11 275 5 Sol: Munchon’s contention is that buying their
=   45
6 6 6 products safeguards health. To weaken that
argument we can show that, for some reason,
08. Rasputin was born in 3233 B.C. The year of their foods might not be healthy.
birth of Nicholas when successively divided So think about an alternative cause
by 25, 21 and 23 leaves remainder of 2, 3
and 6 respectively. If the ages of Nicholas, 10. To open a lock, a key is taken out of a
Vladimir and Rasputin are in arithmetic collection of n keys at random. If the lock is
progression, when was Vladimir born? not opened with this key, it is put back into
(a) 3227 B.C (b) 3229 B.C the collection and another key is tried. The
(c) 3230 B.C (d) 3231 B.C process is repeated again and again. It is
Ans: (c) given that with only one key in the
Solution: The year of birth of Nicholas collection, the lock can be opened. The
25 21 23 probability that the lock will open in ‘n th’
   trail is _____
2 3 6  3227 1
n
 n 1
n

The ages of Nicholas, Vladimir and Rasputin (a)   (b)  


n  n 
are in A.P
 n 1
n n
The ages of Nicholas Vladimir Rasputin 1
(c) 1 –   (d) 1–  
3227 ? 3233  n  n
Nicholas  Rasputin Ans: (c)
 Vladimir age =
2 Sol: Probability that the lock is opened in a trail
3227  3233 1
= = 3230 B.C is (since there is exactly one key, which
2 n
opens the lock)
09. Recent studies have highlighted the harmful  The chance that the lock is not opened in
effects of additives in food (colors, 1
preservatives, flavor enhancers etc.). There a particular trail = 1 –
n
are no synthetic substances in the foods we P(lock is opened in nth trial) = 1– P(lock is
produce at Munchon Foods - we use only not opened in n trials)
natural ingredients. Hence you can be sure
 n  1
n n
 1
you are safeguarding your family’s health = 1 – 1    1  
when you buy our products, says Munchon  n  n 
Foods. Which of the following, if true,
would most weaken the contention of
Munchon Foods?
(a) Some synthetic substances are not
harmful

ACE Engineering Academy Hyderabad |Delhi |Bhopal |Pune |Bhubaneswar |Bengaluru |Lucknow |Patna |Chennai | Vijayawada |Visakhapatnam | Tirupati
:4: Electrical Engg.

One Mark Questions (Q11 – 35) 2(7.7)  13


VLTP  = – 0.2 V
12
11. For the op-amp circuit shown, the op-amp is Hysteresis Voltage = VUTP – VLTP
ideal. The breakdown voltage of zener = 2.566 V
diodes is 7 V and its V = 0.7 V when  2.57 V
forward biased. Then the value of Hysteresis
voltage is ______ (in volts). – 4V

I5 3k
–4V
Vi
3k 10 V –
Vi V0
– +
V0 2k I4
+
2k Vx I1
– 10 V
3k I3 I2 3k
3k 2k

2k
3k 5V
3V
+ 5V
3V –
12. Characteristic equation of a feedback
control system is s3 + ks2 + 8s + 8 = 0, to
obtain fixed amplitude oscillations, the value
Ans: 2.57 range(2.45 to 2.65) of ‘k’ is _____.
Sol: The saturated voltages of the op-amp are Ans: k = 1 (Range = 1)
V0 =  (Vz + V) Sol:
=  7.7 V
When V0 = + 7.7 V, the voltage at non s3 1 8
inverting terminal(Vx) is Vx = VUTP s2 k 8
When V0 = – 7.7 V, Vx = VLTP s1 8k  8 0
Apply KCL at non inverting terminal k
I1 = I2 + I3 + I4 + I5 s0 8 0
V0  Vx Vx  5 Vx  3 Vx Vx  4
   
3k 2k 3k 2k 3k 8k – 8 = 0  k = 1

V0  Vx 3Vx  15  2Vx  6  3Vx  2Vx  8 13. A 400 kV, 3-, 200 km length lossless

3 6 transmission line operated at 50 Hz. If surge
2V0 – 2Vx = 10 Vx – 13 impedance of transmission line is 400 then
12 Vx = 2 V0 + 13 line sending end current at no load is _____
 2V  13  kA.
Vx   0  Ans: 0.123 (Range from 0.10 to 0.13)
 12  Sol: At no load
When V0 = 7.7 V, Vx = VUTP
V
27.7   13 Is  C. sph
VUTP   2.366 V A
12
When V0 = – 7.7 V, Vx = VLTP
ACE Engineering Academy Hyderabad |Delhi |Bhopal |Pune |Bhubaneswar |Bengaluru |Lucknow |Patna |Chennai | Vijayawada |Visakhapatnam | Tirupati
:5: Pre GATE Question Paper with Sol.

1 1 = 19.3411.74
| Is |  sin  . .Vsph
Zc cos   Magnitude of current = 19.34
3 108
 15. If the serial data applied at input x in the
50 following circuit serially from left to right,
1 then the number of times the 1 occurs at z is
 Tan .Vsph
Zc ______.
2
 Z
 D Q D Q D Q
X

= 1  tan 1.047 103  200  180   400 kA Q Q Q


400    3 Clock
= 0.1227 kA.
Data: 10 0 1 1 1 0 0 1 1 1 1 0 1 1 1
14. A volt meter placed across 3 resistor,
shown in above figure reads 45 volts ,then Ans: 4 (Range: 4)
the magnitude of current reading in ammeter Sol: If the input x is 1 in the consecutive clock
is ________ (Amperes) cycles then z becomes 1, means finding the
5
bit pattern ‘111’ and it is finding
10  overlapping pattern.

A 1001110011110111
2
1 2 3 4
V
16. For the circuit shown in figure the diode has
V = 0.7 V, Rf = 20 . The approximate
3 j3 
value of current through the diode is
150 
Ans: 19.34 (Range 19 to 20)
Sol:
10  10/7  +
 10 V
100  100 
A

+ 45 – (a) 25 mA (b) 21 mA
(c) 18 mA (d) None
15 A 3  j3  Ans: (c)
+ V – Sol: The equivalent circuit of the diode is
0.7 V
20 
V = I  Z = 15 (3 + j3) = 45 245 o 
 current through resitive branch
150 
45 245 0.7 V
=  5.56845 o 20 
80 / 7  Vx
10 V
 Total current through Ammeter 100  100 
= 15 + 5.56845

ACE Engineering Academy Hyderabad |Delhi |Bhopal |Pune |Bhubaneswar |Bengaluru |Lucknow |Patna |Chennai | Vijayawada |Visakhapatnam | Tirupati
:6: Electrical Engg.

Apply KCL at Vx VE  6.2  0.6  5.6 V


10  Vx Vx Vx  0.7
  I E =
5 .6
= 56 mA
150 100 120 0 .1 k
10  Vx 12Vx  10Vx  7

15 120 18. In a standard 2nd order system steady state
80 – 8Vx = 22Vx – 7 error to a unit ramp input can be reduced by
30Vx = 87 Vx= 2.9V (a) Increase both n and 
Then current through diode is (b) Increase n and decrease 
V  0 .7 (c) Decrease n and increase 
ID = x  18.33 mA
120 (d) Decrease both n and 
Ans: (b)
17. The zener diode in the figure shown is 2n
temperature compensated and  value of Si Sol: G(s) 
transistor is very high. If the current s (s  2n )
through 0.1 k resistor is 55 mA at 25o C. 
k v  t G(s)  n
What is the approximate current through 0.1 s 0 2
k at 65 C? 1 2
ess    ess  if   n 
10 V k v n

19. If the numbers A = 1 1 1 0 1 1 0 0 and B = 1


0Ω 1 1 1 0 1 1 1 are two signed numbers in 2‘s
0.33 k complement representation, then A – B in
C
2’s complement representation is ________.
B (a) 1 1 1 0 0 0 1 1 (b) 1 1 1 1 0 1 0 1
‘Si’
(c) 0 0 0 1 1 1 0 1 (d) 1 1 1 1 0 1 0 0
Ans: (b)
E Sol:
6.2 V A + 2’s of B
0.1 kΩ

1 1 1 0 1 1 0 0
0 0 0 0 1 0 0 1
1 1 1 1 0 1 0 1
(a) 100 mA (b) 56 mA
(c) 55 mA (d) 54 mA
20. ‘C’ is a cavity in the spherical conducting
Ans: (b)
volume of radius R m. There is a charge ‘q’
Sol: VB = 6.2 V
Coulomb in the cavity as shown in the
VE = VB 0.7 = 5.5 V
figure. spherical conducting volume
5.5
IE = = 55 mA at 25o C
0.1k
surface ‘S’ of the
For 1o C rise in temperature the value of VBE spherical conductor
 by 2.5 mV q
T = 65  25 = 40o C C
R
VBE = 0.7  40  2.5  103 = 0.6 V
VB  VE  0.6

ACE Engineering Academy Hyderabad |Delhi |Bhopal |Pune |Bhubaneswar |Bengaluru |Lucknow |Patna |Chennai | Vijayawada |Visakhapatnam | Tirupati
:7: Pre GATE Question Paper with Sol.

The charge density on the surface ‘S’ is at any point inside the long solenoid is 0 N1
(a) 0 I Wb/m2, along its axis.
(b) Data insufficient Since the short solenoid is along the axis of
q the long one, if it has a radius R m, total flux
(c) C / m2
4R 2
linkages of the short solenoid = 0 N1 I  R2
q N2 Wb-turns (where N2 is the number of
(d) C / m2 turns of the short solenoid). Hence mutual
4R 2

inductance between the two solenoids = 0 


Ans: (c) N1 N2 R2 H.
Sol: Under static conditions, the electric field at Now, if a current ‘I’ flows through the short
any point in the conductor body must be solenoid, flux linkages of the long solenoid
zero. will be 0  N1 N2 R2 Wb-turns.
But q in cavity, by itself, will certainly Substituting given numerical values, flux
produce electric field at all points in the linkages of the long solenoid = 1.024 Wb-
conducting body (and outside). turns.

Hence a charge –q coulomb, drawn from the 22. When primary and secondary windings are
conductor body, appears on the cavity not placed symmetrically with respect to the
surface C. (This is called induced charge). transformer core, the mechanical forces
This leaves charge +q in the conductor developed are
body, which appears on surface S. (a) External radial forces
(b) Internal axial forces
q in cavity and –q on C together produce (c) Internal radial forces
zero field at all points outside C and hence (d) External axial forces
the conductor body. Ans: (d)

q on S must independently produce zero 23. For which value of  the following system
field at all points in the conductor body. For of equations is inconsistent?
this reason, q must distribute uniformly on 3x + 2y + z = 10
surface S. Hence charge density on S  (q/ 2x + 3y + 2z = 10
4R2) c/m2. x + 2 y + z = 10
7 7
(a) (b)
21. A long solenoid has 1000 turns/m and a 5 5
radius greater than (80/ ) cm. A short 5 5
(c) (d)
solenoid of 2000 turns and radius (80/ ) cm 7 7
is located along the axis of the long
solenoid. For a current of 2 A in the short Ans: (a)
solenoid, the total flux linkages of the long 3 2 1
solenoid is.
Sol: 2 3 2  0
(a)  (b) data insufficient
(c) 0 (d) 1.024 Wb-turns 1 2 
Ans: (d)  3(3 – 4) – 2(2–2) + (4–3) = 0
Sol: Assume a current I through the long  5 – 7 = 0
solenoid, and let it have N1 turns/m. Then B 7
=
5
ACE Engineering Academy Hyderabad |Delhi |Bhopal |Pune |Bhubaneswar |Bengaluru |Lucknow |Patna |Chennai | Vijayawada |Visakhapatnam | Tirupati
:8: Electrical Engg.

  27. A 3, 10 kVA load has a p.f of 0.342


24. If X r  cos  r   i sin  r  for r = 1, 2, 3, leading. The power is measured by two
3  3 
.... Then X1. X2. X3 .... (upto infinity) equal wattmeter method. The reading of each
to wattmeter will be
(a) – 1.732 kW & 7.655 kW
(Where, i = 1 ) (b)– 1kW & 4.42 kW
(a)1 + i (b) 1 (c) – 0.577 kW & 2.55 kW
1 3 (d) 1.973 kW & 1.973 kW
(c) i (d)  i
2 2 Ans: (b)
Ans: (c) Sol: Given power is Apparent power
    i r 

S  3 VL I L
Sol: X r  cos  r   sin  r   e  3  ,
3  3  10 103  3 VL I L
r = 1, 2, 3, ... 10 103
1 1 1 
i   2  3 .....  i VL I L 
X1. X2. X3 ...... e 3 3 3 
 e2 = i 3
= 5773.5 VA
25. Inverse Z–Transform of cos = 0.342
Z2  = 70
W1 = VL IL cos (30 + ) = – 1 kW
Z  32 ( Z  2)
W2 = VL IL cos (30 – ) = 4.42 kW
(a) (3n+1 + 2n+1) u(n)
(b) (3n + 2n) u(n) 28.
(c) ((n–2)3n + 2n+1) u(n)
I1 V2 1 I2
(d) None of the above
+
+ +
Ans: (c) R 1
 Z (Z ) 
2 n 1
n 1
Z
V1
Sol: Z–1  V2
 ( Z  3) ( Z  2) 
2
 I1
+
Res/Z=3 +Res/Z=2
= ((n–2)3n + 2n+1) u(n) – –

26. The best resolution of three and half digit


digital multimeter is______. The operating In above circuit, the values of Z22 & Z21
Voltage Ranges are: 0-2V,0-20V,0-200V. respectively are
(a) 2mV (b) 1mV 1 R 1
(a) ,
(c) 3 mV (d) 0.5mV R2 R2
Ans: (b) 1 R 1
(b) ,
Sol: For 3
1 R2 R2
2  R  1  1
(c) ,
1 1
R = N  3  1mV R2 R2
10 10 R 1 1
(d) ,
N = No. Of full digits R2 R2

Ans: (d)

ACE Engineering Academy Hyderabad |Delhi |Bhopal |Pune |Bhubaneswar |Bengaluru |Lucknow |Patna |Chennai | Vijayawada |Visakhapatnam | Tirupati
:9: Pre GATE Question Paper with Sol.

Sol: 30. Consider the cascade of two systems shown


I1 V2 1 in the figure below system ‘B’ is the inverse
I2
+ of system ‘A’
+ +
1 x(t) y(t) System (t) output
LTI System
(b)
V1 I1 R I3 I2 V2 Input (a)
I1 
+
Suppose the input is (t). What is the output
① ② ③
– – (t)?

By applying KVL to mesh ① (a) (t) = u(t); where u(t) is unit step
–V1–V2 + R(I1 – I3) = 0 ........(a) function
(b) (t) = s(t); where s(t); where s(t) is step
By applying KVL to mesh ②
response.
I3 + (I3 + I2) – I1 + R(I3 – I1) = 0 .........(b) (c) (t) = (t)
By applying KVL to mesh ③ (d) (t) = 2u(t) – (t)
–V2+(I2 + I3) – I1 = 0
V2 = I2 – I1 + I3 .........(c) Ans: (c)
From equation (b) Sol: Because the two systmes are inverse, the
I3 
R  1I1  I 2 2nd system should produce the output which
R  2 is same as the initial input.
Substitute value of I3 in equation (c)
Then, V2 = I2 – I1 +
R  1I1  I 2 31. Consider the following LTI systems
R2 x(t) y(t)
h(t)
1 R 1
V2 = I1  I2
R2 R2
 1` R 1 Let x(t) and h(t) be the odd signals. The
 Z21 = , Z 22 
R2 R2 output y(t) is defined as y(t) = x(t) * h(t).
Then the output y(t) is.
29. The short circuit faults are commonly taking (a) Even signal
place in power system. Which of the (b) Odd signal
following statements are correct. (c) Can not be determined
(i) Severe over voltages are taking place an (d) None of the above
unbalanced fault with ground
(ii) In case of unsymmetrical faults, the Ans: (a)
subtransient currents are same as Sol: y(t) = x(t) * h(t) and y(t) = x(–t) * h(–t)
positive sequence subtransient current Because x(t) & h(t) are odd functions
(iii)The pickup value of the relay is selected x(t) = – x(–t) & h(t) = – h(– t).
based on single line to ground fault Substituting eqation (2)
(iv)Positive sequence network terminal y(–t) = [– x(t)] * [– h(t)] = x(t) * h(t) = y(t)
voltage is zero in case of symmetrical  y(t) is even signal.
faults.
(a) (i), (iii) and (iv) (b) (iii) and (iv)
(c) (ii), (iii) and (iv) (d) (i) and (iii)
Ans: (b)
ACE Engineering Academy Hyderabad |Delhi |Bhopal |Pune |Bhubaneswar |Bengaluru |Lucknow |Patna |Chennai | Vijayawada |Visakhapatnam | Tirupati
: 10 : Electrical Engg.

32. A practical diode bridge rectifier with a  di d   R s 


1 
1
filter capacitor is shown below.  dt   L s Ls   i d   
  1     L s v s 
1  vd   
id  d 
dv  0
 dt   c d c d R Load 
+
Ls Rs
+ Cd Rload
33. A 3-phase squirrel-cage induction motor can
s Vd
~
_
be connected to have either 4 or 8 poles. A
 fan operated by this motor takes 15 kW at
low speed. At higher operating speed, the
fan would take _________ kW.
(a) 120 kW (b) 60 kW
(c) 30 kW (d) None
The above circuit can be described by the
following equations in every half cycle of Ans: (a)
line frequency. Sol: For fan load
 di d  T  N 2r ; P  N 3r
 dt  1 
 id   
   A 
 v   LS  VS Nr  N s 
1
 dv d   d  0  Pok
 dt   
3
Then the state matrix A is given by  1 
P  
 LS 1   Pok 
 R 
LS 
(a)  S   15  (8)3 = P2 (4)3
 1 C
 d  P2 = 120 kW
 C d R load 
 RS 1  34. The slip test results of a 3-phase, star
 L 
L 
(b)  S S  connected synchronous machine are given
 1 
1  below
 C d C d R load  Vmax (rms, line-to-line) = 200 V
 1 1  Vmin (rms, line-to-line) = 180 V
 C R Cd 
(c)  d load
 Ia max (rms, phase value) = 20 A
 1 R 
   S Ia min = (rms, phase value) = 15 A
 LS LS 
The direct axis reactance and Quadrature
 RS 1 
 L 
axis reactance per phase respectively
(d)  S Cd
 (a) 13.33 , 9  (b) 5.77 , 6.92 
 1 
1 
(c) 10 , 12  (d) 7.7 , 5.19 
 LS C d R load 

Ans: (d)
Ans: (b)
Vmax / Ph 200 / 3
Sol: From circuit, Sol: Xd =   7.7 
di I min / Ph 15
L s d  R si d  v d  Vs ………(1)
dt Vmin / Ph 180 / 3
Xq =   5.19 
Ia max / Ph 20
Cd dv d  v d  i d ………..(2)
dt R Load

ACE Engineering Academy Hyderabad |Delhi |Bhopal |Pune |Bhubaneswar |Bengaluru |Lucknow |Patna |Chennai | Vijayawada |Visakhapatnam | Tirupati
: 11 : Pre GATE Question Paper with Sol.

35. I Ans: 11 range(10.9 to 11.1)


Network + + Sol: The given circuit is
Z Network
A VA VB B
– – V0
+ I3 Vx
Z = 0 + j1  VA = 12030V 20 Ω 0V 20 Ω I4
VB = 1200V –
20 Ω
In the figure shown above, which of the I1
following statement is correct Vi + I2
– 100 Ω
(a) Power delivered by network A is –
7200.4 W
(b) Power delivered by network B is 7200.4 Apply KCL at inverting terminal
W Vi  0  Vx
  Vx = 5Vi
(c) Power delivered by network A is 7200.4 20 100
W Apply KCL at Vx
(d) Power absorbed by network A is 6800.4  Vx V0  Vx Vx
W  
100 20 20
Ans: (c)  Vx  5V0  5Vx Vx
V  VB 
Sol: I = A 100 20
Z 5V0 6Vx = 5Vx
12030 o  1200 o 5V0 = 11Vx
= = 62.1215 A
j1 5V0 = 11(5Vi)
 The power delivered by network ‘A’ is V0
 11
PA = | VA | | I | cos VA   I  Vi
= (120) (62.12) cos(30 – 15) V0
= 7200.4 W The magnitude of voltage gain  11
Vi
 Power absorbed by network ‘B’ is
PB = |VB||I|cos(VB – I)
37. Consider the system shown in figure below
= (120) (62.12) cos(0 – 15)
= 7200.4 W
 Option ‘C’ is correct R(s) D(s)
k1 (s  2) + k2 C(s)
+– +
Two Mark Questions (Q36 – 65) s3 s  s 4

36. For the op-amp circuit shown, the op amp


assumed to be ideal. The value of voltage
V
gain 0 is _________. The value of ‘k1’ to obtain the steady state
Vi V 0 error of – 0.25 to a unit step disturbance
+
20 Ω  1
input  D(s)   is _______.
20 Ω  s

20 Ω Ans: 6 (Range: 6)
+
– Vi
100 Ω

ACE Engineering Academy Hyderabad |Delhi |Bhopal |Pune |Bhubaneswar |Bengaluru |Lucknow |Patna |Chennai | Vijayawada |Visakhapatnam | Tirupati
: 12 : Electrical Engg.

k 2 Fig. (a) shows a dielectric atom when there


E(s) s(s  4) is no external electric field. Fig. (b) shows
Sol:  the same atom when there is a uniform
D(s) 1  k1k 2 (s  2)
external electric field of 10 ay V/m. The
s(s  3)(s  4)
dipole moment of the atom in fig (b) is
 k 2 (s  3) _________  1028 C-m.
E(s)  D(s)
s(s  3)(s  4)  k1k 2 (s  2)
( k 2 )(3) Ans: 25.6 (range 25 to 26)
Error Lt sE(s)  = – 0.25 Sol: In fig. (a), there is a charge 8q at point N.
s 0 2k1k 2
The electron cloud appears, for points
k1 = 6 outside the sphere, as a charge ‘–8q’ at N.
Thus, net charge at N (for points outside the
38. Open loop transfer function of a unity atom) is zero. The atom is electrically
k neutral. It has no dipole moment.
feedback control system is 4 , the
s  16 In fig. (b) again for points outside the atom,
value of ‘k’ at the break in /away point is there is a charge ‘– 8q’ at C and 8q at N. If
__________. the distance between C and N is y, the
dipole moment is 8qy. Here y = 2  10–9 m
Ans: 16 (Range : 16) and 8q = 12.810–19 C. Dipole moment
Sol: j numerically is 25.6  10 – 28 C-m.

j2 40. In figure, the vector magnetic potential at P


is _________  10 –10 Wb/m.

s=0 z
 z 5m
–2 2 1A

–j2
y=2m P
0

S = 0 is a break point
k s 0  (2) 4  k = 16.
z 5 m
39. Ans: 1924 (Range: 1923 to 1925)
Spherical electron cloud of
radius R m and total charge – 8q C. Sol:
8q
N: Nucleus with charge 8q C. az
N
at center of electron cloud. 
z 5 m
(a) I = 1A 2
dz y  z2
2

z
8q C: Center of the spherical electron cloud,
10 ay N/C N which shifts (with no distortion in shape)
0  P
C
to the left by 1 nano – meter while the nucleus y = 2m
shifts to the right by the same amount.
(b)

q = 1.6  10–19 Coul z 5 m
2
ACE Engineering Academy Hyderabad |Delhi |Bhopal |Pune |Bhubaneswar |Bengaluru |Lucknow |Patna |Chennai | Vijayawada |Visakhapatnam | Tirupati
: 13 : Pre GATE Question Paper with Sol.

z   /2
 0 I dz a z Sol: At rated load, Ea1 = V1 Ia1ra
A(P)    4 y 2  z 2 = 240  50  0.2 = 230 V
z
2 Also Ea1  N11
z   /2
0 I a z dz Rated torque TL1 = Ka1Ia1

4
.   y2  z2
New torque TL2 = 0.6TL1= Ka2Ia2
z
2 Given 1 = 0.96 and 2 = 0.98. [when  is
From figure, z = y tan , dz = y sec2  d mainfield flux at no load]
y 2  z 2  y 2 (1  tan 2 ) = y sec .
TL1 TL1 1 K  0.96    50
z
    a
 0 Ia z 2 TL 2 0.6 TL1 0.6 K a  0.98  I a 2
A(P) 
4  
sec d Ia2 = 29.39 A
z  New counter Emf = Ea2 = 240 – 29.32 (2 +
2

 0 Ia z z
 0.2) = 175.34 V
= log e  sec   tan    2  E a1 N11 230 800  0.96
4 z   
2
E a 2 N 2 2 175.342 N 2  0.98
y 2  ( 2 / 4)  Motor speed, N2 = 597.44 rpm.
z   / 2; sec   , tan  
y 2y
42. A single channel Analog Cathode Ray
Oscilloscope is operated in X-Y Display
 y 2  ( 2 / 4) Mode.An ellipse with major axis in 2nd and
z ; sec   ,
2 y 4th Quadrant and rotating in anti-colckwise
   direction displayed on the screen.The Y-
tan     intercept is at 2 div and Ymax is at 4 div. The
 2y  phase difference between Vertical and
2  Horizontal input signals is_______ (degrees)
y   2
Ia 4 2 Ans: 210 (Range: 210)
A(P)  0 z log e Sol:
4 2 
y2   Y 
4 2  = 180 + sin 1  in 
Substituting given numerical values,  Ymax  Y =2 Y =4 in max

4107 9 5 = 180+30
A(P)  a z log e = 210.
4 9 5
= 1924  10 – 10
Wb/m.
I 800W
700W
41. A 240 V, 50 A, 800 rpm dc shunt motor has 43. 1300 VAR 600 VAR
armature circuit resistanc of 0.2 . If load (Inductive) (Capacitive)
torque is reduced to 60% of its full load +
value and a resistance of 2  is inserted in  120 V 100VAR (Capacitive)

series with armature circuit, the motor speed – (Capacitive)


will be _________ rpm. Armature reaction 80W 120W
1200 VAR
weakens the field flux by 4% at full load and
by 2% at 60% of full load.
The magnitude of ‘I’ in the above figure is
Ans: 597.44 (Range from 597 to 598) ________ Ampere

ACE Engineering Academy Hyderabad |Delhi |Bhopal |Pune |Bhubaneswar |Bengaluru |Lucknow |Patna |Chennai | Vijayawada |Visakhapatnam | Tirupati
: 14 : Electrical Engg.

Ans: 15.025 (Range 14.9 to 15.1) i


Sol: PT = 700 + 800 + 80 + 120 = 1700 W 0.04
QT = 1300 – 600 – 100 – 1200 = – 600 VAR
0.02
ST = PT + jQT =1700–j600=1803–19.4VA
S 1803 0 t
I= T   15.025 A – 0.02
t
V 120
– 0.04
44. – 0.06 i
i
1 2 500 – 0.08
+ – – 0.10
20 V – 0.12
40 V – 0.1053
– 0.5 F
+

45. The number of minimal expressions of the


In the RC circuit shown above, the swich is function f (A, B, C, D) = m(0, 2, 4, 5, 6, 7,
closed on position 1 at t = 0 and after 1 time 8, 10, 12, 13, 15) in sum of products form is
constant is moved to position 2. _______.
Then the value of the current just after
switch is moved to position 2 is _______ Ans: 4 (Range: 4)
Ampere Sol:
Ans: – 0.1053 (Range – 0.1050 to – 0.1060)
Sol: When switch is at position 1: CD
AB 00 01 11 10
i = C1e–t/RC = C1e–4000t
00 1 1
RC = 500  0.5  10–6 = 250 10–6
V 20
 At t = 0, i0 =   0.04 A 01 1 1 1 1
R 500
 i = 0.04 e–4000t 11 1 1 1
One time constant = 1RC = 500  0.5  10–6
10 1 1
= 250 micro seconds
At this point i = 0.04e–1 = 0.0147 Amp
When switch is moved to position 2:
Voltage across capacitor c = 20(1– e-1) = B D  BD  BC  AB
12.65 V Vc (0  )   Vc 0   (or)
This voltage and the 40 Volt source both BD  BD  BC  A D
drive current in the opposite direction from
(Or)
the current caused by 20 volt source.
The current equation for the second transient BD  BD  CD  A B
is i = C2 e–4000(t – t) (Or)
 40  12.65 BD  BD  CD  A D
 At t = t, i =
500
= –0.1053 Amp

 i = – 0.1053 e–4000(t –t)

ACE Engineering Academy Hyderabad |Delhi |Bhopal |Pune |Bhubaneswar |Bengaluru |Lucknow |Patna |Chennai | Vijayawada |Visakhapatnam | Tirupati
: 15 : Pre GATE Question Paper with Sol.

46. A 3-phase WRIM develops rated torque at a { [ky – 3x2y – y3] i + [kx – x3 – 3xy2] j }.
slip of 0.05 with slip ring short circuited. If
 i j  15
the rotor circuit resistance is increased to    
four times by inserting external resistance in  2 2  2
the rotor circuit, then at rated load torque, i j 
the slip would be ______.  2k  6  8  i   k  1  12  j  .   
 2 2
Ans: 0.2 (Range: 0.2) 15
=
Sol: For fan load 2
SV 2 k=4
T ;
R2
49. An inverter employs output voltage matches
KSV 2
T to reduce it’s lower order harmonics. The
R2 output wave form is as shown in the
S following figure. The rms value of output
For the same torque, = Constant
R voltage V0 at fundamental frequency is 70 V
S1 S for 1 = 30. Then 2 ________ degrees.
 2 V0
R 21 R 22
0.05 S2 100 V
 1
R 4R
S2 = 0.05  4 = 0.2 2
/2  2 t

2
47. The value of  2
| 1  x 4 | dx  ________ –2
–1
Ans: 12 (Range: 12)
2 2
Sol: 2
| 1  x 4 | dx  2 | 1  x 4 | dx ( | 1  x 4 | is
0
Ans: 41 (Range: 39 to 43)
Sol: The fourier analysis of the wave form is to
even function) be carried out to find out fundamental
1
 
= 2{ 1  x 4 dx   1  x 4 dx}
0 1
2
  component. The wave form is having
quarter wave symmetry.
= 12
Hence An = 0

Bn =   sin nt dt 


48. Let f (x,y) = k xy – x3y – xy3 for (x, y)  R2, 4
where ‘k’ is a real constant. The directional 
derivative of f(x,y) at the point (1,2)
 int the 4 1
 2 /2 
i j 15   100 sin nt dt     100sin nt dt   100 sin nt dt 
  0
direction of unit vector  is . 1 /2 
2 2 2
Then the value of k is ________________. 4  100 1  2 cos n1  2 cos n 2 

  n 

Ans: 4 (Range: 4)
Sol: (grad f). â 
15 B1 =
400
1  2 cos 1  2 cos  2 

2
 70 2  B1 
400
1  2 cos 30  2 cos  2 

2 = 41
ACE Engineering Academy Hyderabad |Delhi |Bhopal |Pune |Bhubaneswar |Bengaluru |Lucknow |Patna |Chennai | Vijayawada |Visakhapatnam | Tirupati
: 16 : Electrical Engg.

50. Consider the following two bus power 52. The system described by the differential
system network with complex power d2 y dy du
demands at buses 1 and 2 as SD1= 1 + j1 equation 2
 7  12y  5  17u . If
dt dt dt
p.u; and SD2 = 1 + j1 p.u respectively. Bus-2 the state space representation is in diagonal
voltage is given as 1 p.u in magnitude. canonical form, given by
  Ax(t)  Bu(t) and Y(t) = C X(t). If
x(t)
Bus - 1 Bus - 2
Z = j 0.2 p.u matrix C  [0.5 0.5] . Then matrix ‘B’ is
 2  3
(a)   (b)  
6 6
SD1 = 1 + j1 SD2 = 1 + j1 4 6
(c)   (d)  
6  3
The angle made by Bus-1 voltage with Ans: (c)
respect to Bus-2 voltage will be ______ Y(s) 5s  17 5s  17
degrees. Sol: T.F = = 2 =
U(s) s  7s  12 (s  3)(s  4)
Ans: 9.46 (Range: 9 to 10) b b
Sol: T.F  1  2
s3 s4
Bus-1 Bus-2 b1 = 2 , b 2 = 3
Z = j0.2
 4
V1
I2
I2
 B 
6
SD1 = 1 + j1 SD2 = 1 + j1
53. The minimum number of two input NOR
Let V2 = 1.00 p.u gates required to implement function
as S2 = V2. . I2 f (A, B, C, D) = m (1, 3, 4, 7, 8, 9, 11, 12,
1  j1 14, 15) additionally with the following 4  1
I2  mux are _______ (assuming only
10
I2 = 1 – j1 pu uncomplemented variables are available).
now Bus-1 voltage, V1 = V2 + I2 Z
V1 = 10 + (1 – j1) (j0.2) I0
= 1.2169.46
I1 4  1 y f (A, B, C, D)
I2 mux
–4t
51. Unit step response of a system is 2(1 – e )
u(t) then the final value of the output of the I3
system to a unit impulse is _____. S1 S0
(a) 0 (b) 2
(c) 4 (d) None
Ans: (a) C D
d
Sol: IR   2(1  e 4t )u(t)  = 8 e–4(t) u(t) (a) 6 (b) 5
dt (c) 3 (d) None
t IR  0
t 

ACE Engineering Academy Hyderabad |Delhi |Bhopal |Pune |Bhubaneswar |Bengaluru |Lucknow |Patna |Chennai | Vijayawada |Visakhapatnam | Tirupati
: 17 : Pre GATE Question Paper with Sol.

Ans: (b) (1732  0.8  19.2  0.2) 2


Sol: =
 (1732  0.6  19.2  2.99) 2
C D I0
I1 I2 I3
= 1701.3 V
S1 S0 00
01 10 11
Regulation =
A B(0 0) 0 2 EV 1699  1730
A B(01) 5 6  100   100  1.8%
V 1730
AB(10) 10
AB (1 1) 13 55. A balanced three – phase AC voltage source
is connected to a balanced star connected
A+B B AB 1 load through a star-delta transformer as
shown in the figure. If the magnetizing
current is neglected and Ia = 25 A and Ib =
A
B I0 500. The line-to-line voltage is 20060
I1 4  1 y f (A, B, C, D)
on the star side, then what is the value of ‘V’
B
mux in figure (in volts).
A I2 Ia Ib
1 I3
S1 S0 R
V=?
C D

54. A 3- , 100 kVA, 3000 V star connected


alternator has an effective armature
resistance is 0.2 . A field current of 40 A R
produces a short circuit current of 200 A and
open circuit emf of 1040 V (line to line).
The full load regulation for 0.8 leading
power factor is
(a) 2.2% (b) 1.8% R
(c) 4.833% (d) 81.66%
Ans: (b)
1040
Sol: VL = 1040 V  Vph = = 600 V
3 (a) 10060
100 103 100
Ifull =  19.2A ; (b) 60o
3  3000 3
100
Vph =
3000
 1730 V (c) 30o
3 3
E 600 (d) 10030
Zs = OC   3  , Ra = 0.2 
Isc 200
Xs = Zs2  R a2 = 32  0.22 = 2.99 

 V cos   Ia R a   (V sin   I a X s ) 2
2
E=

ACE Engineering Academy Hyderabad |Delhi |Bhopal |Pune |Bhubaneswar |Bengaluru |Lucknow |Patna |Chennai | Vijayawada |Visakhapatnam | Tirupati
: 18 : Electrical Engg.

Ans: (d) 1500  103


Sol: I  32.80  36.86o
3  33  10  0.8
3

2530 500 33k 33k


x  32.8036.86
50 + 3 3
200
30o 30o R 50R0
3 3 – ×(1.5+j24+3(0.5+j6.5)
20060 50 3R30 o 33k
x  20016.463.100
3
x = 1.0505
50–120 x 1
+
R 50R–120 %tap =  100  5.05%
– 1

1
57. If the probability of hitting a target is and
5
if 10 shots are fired, what is the conditional
R
probability that the target being hit atleast
twice assuming that atleast one hit is already
50 scored?
Vph1 I ph 2 (a) 0.6999 (b) 0.624
  3 (c) 0.892 (d) 0.268
Vph 2 I ph1 25
Ans: (a)
Px  2  1  q n  npq n 1
Sol: P(x  2| x  1) = =
Px  1
Vph2 = 10030
1 qn
56. A 3-phase load of 1500 kW, 0.8 power =
10 9
factor lagging is supplied from a tap-changer 4  1  4 
transformer, 11/33 kV, delta on LV and star 1     10  
5  5  5  = 0.6999
on HV side. The leakage impedance of LV 10
4
winding is (0.5 + j6.5) per phase and that 1  
of H.V. windings is (1.5 + j24) . If the low 5
voltage side is supplied at 11 kV, find the
setting of tap-changer to give 33 kV at full 58. Let w (y1, y2) be the wronskian of two
load. linearly independent solutions y1 and y2 of
(a) 5.05% (b) 6.05% the equation, y  p(x) y  Q(x) y  0 . The
(c) 4.05% (d) 3.05% product of w(y1, y2).p(x) =
(a) y 2 y1  y1 y2 (b) y1 y1  y 2 y2
Ans: (a) (c) y1 y2  y2 y1 (d) y1 y2  y1y2
Sol:
Ans: (a)
1.5 j24 (0.5+j6.5)k2 Sol: y1  p(x)y1  Q(x)y1  0.......(1)
+ + y2  p(x)y2  Q(x)y 2  0.......(2)
33k 33kv
3
x
3 (1)  y2 – (2) y1  p(x)
 y1 y2  y1y2   y2 y1  y1y 2
– –
 w (y1, y2) p(x) = y 2 y1  y1 y2

ACE Engineering Academy Hyderabad |Delhi |Bhopal |Pune |Bhubaneswar |Bengaluru |Lucknow |Patna |Chennai | Vijayawada |Visakhapatnam | Tirupati
: 19 : Pre GATE Question Paper with Sol.

59. Given the differential equation y1 = x – y Sol: The total admittance is


with initial condition y(0) = 0. The value of 1 1
YT = 
y(0.1) calculated numerically upto the third R L  j10 4  j5
place of decimal by the 2nd order Runge-
 R 4  5 10 
Kutta method with step size h = 0.1 is =  2 L    j  2 
_____________  R L  100 41   41 R L  100 
 For resonance imaginary part must be
Ans: 0.005 (Range: no need) zero
Sol: Given y1 = x – y 5 10
Also given y(0) = 0 and h = 0.1 i.e,  2 , R 2L  18
41 R L  100
y(0.1) = ?
Let x0 = 0, x1 = x0 + 1.h = 0 + 0.1 = 0.1  Thus there is no value of RL which results
The 2nd order Runge-Kutta method is given in resonacne.
by
1 61. Synchronous generator is connected in an
y1 = y(x1) = y0 + (k1 + k2) infinite bus through a transformer and
2
where k1 = h f(x0, y0) and double circuit line. The infinite bus voltage
k2 = h f(x0 + h, y0 + k1) is 1.0 pu and the internal voltage of
generator is also 1.0 pu. The tansfer
k1 = (0.1) [x0  y0] = (0.1) (0 – 0) = 0
reactance before fault is 0.5 pu. A 3-phase
k2 = (0.1) [(x0 + h) – (y0 + k1)]
fault is taking place near to the bus bar on
= (0.1) [0 + 0.1 – (0 + 0)] = 0.01
one of the double circuit. The fault is
1 0.01
y1 = y(0.1) = 0 + (0 + 0.01) = cleared by circuit breaker after few cycles.
2 2 The peak power after fault cleared is 75% of
= 0.005 peak power before fault. The critical
clearing angle of the rotor at the fault
60. cleared by CB is when the generator is
delivery 1.0 pu power before fault to infinite
4
bus.
RL
(a) 68.5o (b) 74.8o
(c) 59.2o (d) 52.6o
j10 
Ans: (c)
– j5 
EV 1.0  1.0
Sol: Pm1 =   2 .0
X eq 0 .5
For the circuit shown above, which of the Pm2 = 0
following statement is true Pm3 = 0.75 Pm1 = 1.5
(a) The resonance is possible when RL = 4   PS 
0 = sin 1   = 30, 0(rad) = 0.523
only
 Pm1 
(b) The resonance is possible when RL = 10
 only  PS 
m = 180  sin 1  
(c) The resonance is possible for all positive  Pm3 
values of RL  1.0 
(d) The resonance is not possible with any = 180  sin 1   = 138.2
o

value of RL  1.5 
Ans: (d) m(rad) = 2.41

ACE Engineering Academy Hyderabad |Delhi |Bhopal |Pune |Bhubaneswar |Bengaluru |Lucknow |Patna |Chennai | Vijayawada |Visakhapatnam | Tirupati
: 20 : Electrical Engg.

 PS(m  )  Pm3 cos  m 


1
z112 = j10, y112 = j0.1
c = cos  
 Pm3  z20 = j0.5, y20 = j2
y33 = y30 + y30(2) + y31 = j15.25
1.0  2.41  0.523)  1.5cos138.2  
= cos 1   z23 = j0.1, y23 = j10
 1.5  y12 = y12 = (j5) = j5
1.0  2.41  0.523  1.118  z30 = j0.25, y30 = j4
= cos 1   z30(2) = j0.8, y30(2) = j1.25
 1.5 
y12 = y21
= cos  0.512  = 59.2
1 o
y13 = 0 = y31
y23 = j10 = y32
62. A power system network is shown below
along with per unit reactances. Bus 63. Find the energy of the signal
admittance matrix is x(t) = 8 sinc (4t) cos 2t
(a) 12 J (b) 8 J
① ② ③
j 0.5 j0.2 j0.1 j0.25 6
~ (c) J (d) None of the above
-j10 
j 0.4 j0.5 j0.8 Ans: (a)
~ Sol: 8 sinC 4(t)  2 rect (f/4)
Using frequency shifting property
 7 5 10 
 f 1   f 1
(a) j  5 17 10 
 x(f )  rect    rect  
 4   4 
10 10 14  
Energy E   | x t  |2

3
 7 5 0  =  x 2 (t)
(b) j  5  17 10 
 3
1
 0 10  14
1 3
 1  2
 4  1 = 2 + 8 + 2
 9.45 5 0  3 1 1

(c) j  5 16.95 10  = 12 J
 0 10 15.25 64. A AC voltage controller fed from 230 V, 50
 9.5 5 15  Hz AC supply and supplying power to an

(d) j  5 17 10  inductive load of R = 100 , L = 0.318 H.
The output across the load is alternating and
 15 10 15.25 switching devices are given with firing angle
Ans: (c) of 30 in every half cycle. The rms, average
Sol: z10 = j0.5, y10 = j2 voltages across load are
y112 (a) 230 V, 0 V
y11 = y10 + y10(2) + y12 + = j9.45 (b) 226.65 V, 179.3 V
2
z10(2) = j0.4, y10(2) = j2.5 (c) 226.65 V, 0 V
(d) Data insufficient
z12 = j0.2, y12 = j5
y1
y22 = y20 +y21 +y23 + 12 = j16.95
2

ACE Engineering Academy Hyderabad |Delhi |Bhopal |Pune |Bhubaneswar |Bengaluru |Lucknow |Patna |Chennai | Vijayawada |Visakhapatnam | Tirupati
: 21 : Pre GATE Question Paper with Sol.

Ans: (a) Ans: (a)


Sol: Given circuit is AC voltage controller whose Sol: VL
outout is AC.
R = 100 
XL = 2fL = 2×0.318 ×50 = 100 
X   100 
= tan 1  L  = tan 1   = 45
Vs = 100 V T
 R   100  t
TON V0 = 100 V
Since  < 45; then rms output voltage is
supply voltage.
Vor = 230 V ; V0 = 0 Given circuit is Buck Boost converter
voltage across inductor would be
65. A DC-DC converter is used to feed the VS = 100 V
resistive load and its output is always V0 = 100 V
inverted. An alternating square wave D
voltage is getting produced across inductor V0 = VS 
having rms value of 100 V and frequency of 1 D
50 Hz. The duty cycle of the switch is D
100 = 100×
_________ 1 D
1
D=1D=D= = 0.5
(a) 0.5 2
(b) 0
(c) Alternating voltage can not be produced
in DC- DC converter
(d) Can not be determined

ACE Engineering Academy Hyderabad |Delhi |Bhopal |Pune |Bhubaneswar |Bengaluru |Lucknow |Patna |Chennai | Vijayawada |Visakhapatnam | Tirupati

Вам также может понравиться